Solve: x - 1 < 3 help me plssss

Answers

Answer 1

Answer:

x =2

Step-by-step explanation:

becaue 2-1 is smaller than 3

Answer 2

Answer:

Hello!

I believe your answer is:

x=2

If this is not correct, please let me know and I will try again!

Step-by-step explanation:


Related Questions

What’s the correct answer for this question?

Answers

Answer:

the radius

Step-by-step explanation:

the correct answer is the radius

If 9: x= x-4, then x=
0 36
18
0 24
6

Answers

Answer:

2±√13

Step-by-step explanation:

9/x=x-4

x² -4x - 9=0

x² -4x +4- 13=0

(x -2)²=13

x-2= ±√13

x= 2±√13

In a survey, participants were asked how much confidence they had in the economy.
The results were as follows:



Response Number

A great 3,187
deal

Some
9,120

Hardly 5,149
any



What is the probability that a sampled person has either some confidence or a great
deal of confidence in the economy? Write only a number as your answer. Round to
two decimal places (for example: 0.43). Do not write as a percentage.

Answers

Answer:

0.71

Step-by-step explanation:

Great Deal or Some = 12,307

Total Participants = 17,456

Probability = 12,307/17,456 = 0.71

A tree grows three feet per year. What happens to the growth of the
When the number of years increases, the number of feet decrea
When the number of years decreases, the number of feet stays
When the number of years increases, the number of feet increas
When the number of years decreases, the number of feet increa

Answers

Answer:

The answer is C :,)

Step-by-step explanation:

Answer:

The answer to your question is c

Step-by-step explanation:

Because the years have to increase for it to grow.

One third of the sum of 15
and thrice a certain number is
equal to twice the number. Find
the number​

Answers

Answer:

x=-1/39

Step-by-step explanation:

Please answer this correctly without making mistakes

Answers

Answer:

589

Step-by-step explanation:

l x w

19x11

5x31

5x45

589

Answer:

589 is the answer

The Tennessean, an online newspaper located in Nashville, Tennessee, conducts a daily poll to obtain reader opinions on a variety of current issues. In a recent poll, readers responded to the following question: "If a constitutional amendment to ban a state income tax is placed on the ballot in Tennessee, would you want it to pass?

Required:
a. What was the sample size for this poll?
b. Are the data categorical or quantitative?
c. Would it make more sense to use averages or percentages as a summary of the data for this question?
d. Of the respondents, 67% said Yes, they would want it to pass. How many individuals provided this response?

Answers

Answer:

Answers below

Step-by-step explanation:

a. What was the sample size for this poll?

b. Are the data categorical or quantitative?

c. Would it make more sense to use averages or percentages as a summary of the data for this question?

d. Of the respondents, 67% said Yes, they would want it to pass. How many individuals provided this response?

Please help!! Which of the following is equal to the rational expression when x ≠ 2 or -4? 5(x-2)/(x-2)(x+4)

Answers

Answer:

5 / (x+4)      x ≠2 x≠-4

Step-by-step explanation:

5(x-2)/(x-2)(x+4)

The denominator cannot be zero so x ≠2 x≠-4

Cancel like terms in the numerator and denominator

5 / (x+4)      x ≠2 x≠-4

A car travelled 80km in 48minutes. find the speed of the car in km/hr​

Answers

80km / 48 min = 1 2/3 km per minute.

1 2/3 km per minute x 60 minutes(1 hour) = 100 km per hour

85 points!! | All of the following expressions have the same value, when x= -2 and y= 4, except
-2xy
0-4x2
0x²y
0 (-2) ²y

Answers

Answer:

They have two sets of equal answers...

Step-by-step explanation:

-2 * 2 * 4 = -16

0 - 4 * -2 * -2 = -16

0 * -2 * -2 * 4 = 0

0 * 4 * 4 = 0

Find lim x→3 sqrt 2x+3-sqrt 3x/ x^2-3x. you must show your work or explain your work in words plsss I need help

Answers

I'm assuming the limit is supposed to be

[tex]\displaystyle\lim_{x\to3}\frac{\sqrt{2x+3}-\sqrt{3x}}{x^2-3x}[/tex]

Multiply the numerator by its conjugate, and do the same with the denominator:

[tex]\left(\sqrt{2x+3}-\sqrt{3x}\right)\left(\sqrt{2x+3}+\sqrt{3x}\right)=\left(\sqrt{2x+3}\right)^2-\left(\sqrt{3x}\right)^2=-(x-3)[/tex]

so that in the limit, we have

[tex]\displaystyle\lim_{x\to3}\frac{-(x-3)}{(x^2-3x)\left(\sqrt{2x+3}+\sqrt{3x}\right)}[/tex]

Factorize the first term in the denominator as

[tex]x^2-3x=x(x-3)[/tex]

The [tex]x-3[/tex] terms cancel, leaving you with

[tex]\displaystyle\lim_{x\to3}\frac{-1}{x\left(\sqrt{2x+3}+\sqrt{3x}\right)}[/tex]

and the limand is continuous at [tex]x=3[/tex], so we can substitute it to find the limit has a value of -1/18.

The lengths of a professor's classes has a continuous uniform distribution between 50.0 min and 52.0 min. If one such class is randomly selected, find the probability that the class length is between 50.1 and 51.1 min. P(50.1 < X < 51.1) =

Answers

Answer:

P(50.1 < X < 51.1) = 0.5

Step-by-step explanation:

An uniform probability is a case of probability in which each outcome is equally as likely.

For this situation, we have a lower limit of the distribution that we call a and an upper limit that we call b.

The probability that we find a value X between c and d is given by the following formula:

[tex]P(c < X < d) = \frac{d - c}{b - a}[/tex]

The lengths of a professor's classes has a continuous uniform distribution between 50.0 min and 52.0 min.

This means that [tex]a = 50, b = 52[/tex]

So

[tex]P(50.1 < X < 51.1) = \frac{51.1 - 50.1}{52 - 50} = 0.5[/tex]

Every product manufactured by a company goes through 6 different tests before being shipped out. It is known that the probability that a product passes any single test is 0.9 and the tests are independent. Only those products that pass the first three tests and also pass at least one of the three remaining tests are shipped out. Find the probability that a manufactured product is shipped out.

Answers

Answer:

The Probability that the product is shipped out is 0.7283

Step-by-step explanation:

Here, we are given that, a product passes through 6 tests before it is shipped out and a product is shipped out only if it passes all the first 3 tests and at least 1 of the remaining 3 tests.

We have P(pass)= 0.9, is the Probability of passing any test.

Which implies, P(fail)= 1- 0.9= 0.1

We have to find the Probability that the product is shipped out.

P(product is shipped out) = P(it passes first 3 tests )*P(passes at least 1 of the remaining 3 tests) •••••••••••(i)

We can take the product as the tests are Independent.

Now, let us obtain

P(it passes first 3 tests ) = P(pass)*P(pass)*P(pass)

=P(pass)]^3 = (0.9)^3 = 0.729

Hence, P( it passes first 3 tests)= 0.729 •••••••(ii)

Now,

P(passes at least 1 of the remaining 3 tests)

= 1-P(fails all the 3 remaining tests)

= 1-(0.1)^3 = 1 - 0.001 = 0.999

Hence,

P(passes atleast 1 of the remaining 3 tests)=0.999 ••••••••(iii)

Now, substituting the 2nd and 3rd equations in the first equation, we have;

P(product is shipped out) = P(it passes first 3 tests )*P(passes at least 1 of the remaining 3 tests)

= (0.729)*(0.999)

= 0.728271

= 0.7283

ACB = DCE
A = 3x-10, C = 45°, D = 2x+10
Please help confused

Answers

Answer:

  x = 20

Step-by-step explanation:

The congruence statement tells you that angle A is congruent to angle D. (Both are listed first in the triangle names.) This means ...

  ∠A = ∠D

  3x -10 = 2x +10

  x = 20 . . . . . . . . . . add 10-2x to both sides

The top of a lighthouse is 100 m above sea level. The angle of elevation from the
deck of the sailboat to the top of the lighthouse is 28°. Calculate the distance
between the sailboat and the lighthouse.

Answers

Answer:

  188 m

Step-by-step explanation:

The tangent of the angle is the ratio of the side opposite (height of the lighthouse) to the side adjacent (distance to the lighthouse):

  tan(28°) = (100 m)/distance

  distance = (100 m)/tan(28°) ≈ 188 m

The distance between the sailboat and the lighthouse is about 188 m.

Triangles E F G and K L M are shown. Angles E F G and K L M are congruent. The length of side K L is 6, the length of side M L is 5, and the length of K M is 8. The length of E G is 24, the length of G F is 15, and the length of E F is 18. Can the triangles be proven similar using the SSS or SAS similarity theorems? Yes, △EFG ~ △KLM only by SSS. Yes, △EFG ~ △KLM only by SAS. Yes, △EFG ~ △KLM by SSS or SAS. No, they cannot be proven similar by SSS or SAS.?

Answers

Answer:

The Answer is C: Yes, △EFG~ △KLM by SSS or SAS

Step-by-step explanation:

SSS is for side-side-side

Both triangles have all three sides given, so the SSS similarity theorem is one way to prove these triangles are similar.

SAS is for side-angle-side

Both triangles have one angle measurement given, and two side lengths given, therefore we can also use the SAS similarity theorem to prove the two triangles are similar.  

Since both SSS and SAS work to prove the triangles are similar, the correct answer is C: Yes, △EFG~ △KLM by SSS or SAS

(I also just answered this question on the assignment and got it correct)

Answer:

Answer is C

Step-by-step explanation:

Took it on Edg

Complete the statements with equal to, greater than, or less than. 5 6 × 6 9 is ? 5 6 . 6 × 5 6 is ? 5 6 . 5 6 × 9 9 is ? 5 6 . 5 6 × 8 7 is ? 5 6 . 7 7 × 5 6 is ? 5 6 . 5 6 × 5 6 is ? 5 6 .

Answers

Answer:

someone already answered

Step-by-step explanation:

srry

Use Newton's method with initial approximation x1 = 1 to find x2, the second approximation to the root of the equation x4 − x − 3 = 0. (Round your answer to four decimal places.) x2 =?

Answers

Answer:

[tex]x_{2} = 0.0000[/tex]

Step-by-step explanation:

The formula for the Newton's method is:

[tex]x_{i+1} = x_{i} + \frac{f(x_{i})}{f'(x_{i})}[/tex]

Where [tex]f' (x_{i})[/tex] is the first derivative of the function evaluated in [tex]x_{i}[/tex].

[tex]x_{i+1} = x_{i} + \frac{x_{i}^{4}-x_{i}-3}{4\cdot x_{i}^{3}-1}[/tex]

Lastly, the value of [tex]x_{2}[/tex] is determined by replacing [tex]x_{1}[/tex] with its numerical value:

[tex]x_{2} = x_{1} + \frac{x_{1}^{4}-x_{1}-3}{4\cdot x_{1}^{3}-1}[/tex]

[tex]x_{2} = 1.0000 + \frac{1.0000^{4}-1.0000-3}{4\cdot (1.0000)^{3}-1}[/tex]

[tex]x_{2} = 0.0000[/tex]

Calculating a correlation can help describe a relation between two quantitative variables' ___ and ___ . However, it is not sufficient to use a correlation coefficient to describe two variables. The addition of ___ can provide other helpful details such as __ _."

Answers

Answer:

direction

shape

scatter plots

shape and outliers

Step-by-step explanation:

Correlation is defined as the degree of correspondence between two variables.

When the values increase together, correlation is positive and when one value decreases as the other increases, correlation is negative .

Calculating a correlation can help describe a relation between two quantitative variables' direction and shape. However, it is not sufficient to use a correlation coefficient to describe two variables.  The addition of scatter plots can provide other helpful details such as shape and outliers

What’s the correct answer for this question?

Answers

Answer:

Height = 12 inches

Step-by-step explanation:

Volume = Area × Height

1080 = 90 × H

H = 1080/90

H = 12 inches

Engineers must consider the breadths of male heads when designing helmets. The company researchers have determined that the population of potential clientele have head breadths that are Normally distributed with a mean of 6.5 inches and a standard deviation of 1.2 inches. According to the 68-95-99.7 rule, we expect 95% of head breadths to be:___________.

Answers

Answer:

"According to the 68-95-99.7 rule, we expect 95% [95.45%] of head breadths to be" between 4.1 inches and 8.9 inches.

Step-by-step explanation:

According to the 68-95-99.7 rule, approximately:

68% (more precisely, 68.27%) of the data from the normal distribution lie one standard deviation, [tex] \\ \sigma[/tex], above and below the population mean, [tex] \\ \mu[/tex].95% (more precisely, 95.45%) of the data lie two standard deviations, [tex] \\ 2\sigma[/tex], above and below the population mean, [tex] \\ \mu[/tex], and finally,99.7 (or more precisely, 99.73%) of the data lie three standard deviations, [tex] \\ 3\sigma[/tex], above and below the population mean, [tex] \\ \mu[/tex].

Then, if we have--from the question--that:

The random variable is head breadths.This variable follows a normal distribution.The population's mean for this distribution is [tex] \\ \mu = 6.5[/tex] inches.The population's standard deviation is [tex] \\ \sigma =1.2[/tex] inches.

We have to remember that two parameters characterize a normal distribution: the population's mean and the population's standard deviation. So, mathematically, the distribution we have from question is [tex] \\ N(6.5, 1.2)[/tex].

For 95% (95.45%) of the head breadths, we expect that they are two standard deviations below and above the population's mean.

For solving this, we need to use the cumulative standard normal distribution (in case we need to find probabilities) and also use standardized values or z-scores:

[tex] \\ z = \frac{x - \mu}{\sigma}[/tex] [1]

A z-score "tells us" the distance from the mean in standard deviations units. If the z-score is positive, it is above the mean. If it is negative, it is below the mean.

Since 95% (95.45%) of the head breadths are two standard deviations (above and below the mean), we have (using [1]):

[tex] \\ \pm2 = \frac{x - \mu}{\sigma}[/tex]

But we already know that [tex] \\ \mu=6.5[/tex] inches and [tex] \\ \sigma=1.2[/tex] inches.

Thus (without using units) for values above the population's mean:

[tex] \\ 2 = \frac{x - 6.5}{1.2}[/tex]

Solving the equation for x, we multiply by 1.2 at each side of [1] :

[tex] \\ 2 * 1.2 = \frac{x - 6.5}{1.2} * 1.2[/tex]

[tex] \\ 2 * 1.2 = (x - 6.5)\frac{1.2}{1.2}[/tex]

[tex] \\ 2 * 1.2 = (x - 6.5)\frac{1.2}{1.2}[/tex]

[tex] \\ 2 * 1.2 = (x - 6.5)*1[/tex]

[tex] \\ 2 * 1.2 = x - 6.5[/tex]

Adding 6.5 at each side of the previous equation:

[tex] \\ (2 * 1.2) + 6.5 = x - 6.5 + 6.5[/tex]

[tex] \\ (2 * 1.2) + 6.5 = x + 0[/tex]

[tex] \\ (2 * 1.2) + 6.5 = x[/tex]

Therefore, the raw value, x, in the distribution that is two standard deviations above the population's mean is:

[tex] \\ x = (2 * 1.2) + 6.5[/tex]

[tex] \\ x = 2.4 + 6.5[/tex]

[tex] \\ x = 8.9[/tex] inches.

For two standard deviations below the mean, we proceed in the same way:

[tex] \\ -2 = \frac{x - 6.5}{1.2}[/tex]

[tex] \\ -2*1.2 = x - 6.5[/tex]

[tex] \\ (-2*1.2) + 6.5 = x[/tex]

[tex] \\ x = (-2*1.2) + 6.5[/tex]

[tex] \\ x = -2.4 + 6.5[/tex]

[tex] \\ x = 4.1[/tex] inches

Therefore, "according to the 68-95-99.7 rule, we expect 95% [95.45%] of head breadths to be" between 4.1 inches and 8.9 inches.

The graph below shows these values, and the shaded area represents 95% of the data, or, to be more precise, 95.45% (0.954499).  

Could someone please help me with the steps for this problem? Factor by grouping: w²+3w+w+3

Answers

Answer:

Please see steps below

Step-by-step explanation:

In order to factor by grouping, we divide the four terms given into two groups, and extract on each group any common factor  we can.

In our example, we can select the terms: [tex]w^2[/tex]  and [tex]w[/tex] as one of our groups, and [tex]3w[/tex] and 3 in the other group. Then we re-organize the expression as:

[tex](w^2+w)+(3x+3)[/tex]

Now we extract from the first binomial group, the factor [tex]w[/tex] as a common factor of both terms, and from the second group we extract the factor "3" as common factor of those two terms:

[tex](w^2+w)+(3x+3)\\w(w+1)+3(w+1)[/tex]

We notice now that after the extraction, we are left with two exactly equal binomial factors [tex](w+1)[/tex] that appeared in the first group and in the second group. We proceed then to extract it as common factor for the two groups:

[tex]w(w+1)+3(w+1)\\(w+1)(w+3)[/tex]

this last product of two binomials ([tex](w+1)\,(w+3)[/tex] is the result of factoring the original expression.

solve 5(x+4)<75 sdsdsd

Answers

Answer:

x < 11

Step-by-step explanation:

[tex]5(x+4)<75 \\ open \: the \: bracket \: using \: 5 \\ 5x + 20 < 75 \\ subtract \: - 20 \: from \: both \: sides \: [/tex]

[tex]5x + 20 - 20 < 75 - 20 \\ 5x < 55 \\ divide \: both \: sides \: of \: the \: equation \: \\ by \: 5[/tex]

[tex] \frac{5x}{5} < \frac{55}{5} \\ x < 11[/tex]

The required solution of inequality is,

⇒ x < 11

We have to simplify the expression,

⇒ 5 (x + 4) < 75

We can simplify it by definition of inequality as,

⇒ 5 (x + 4) < 75

⇒ 5x + 20 < 75

Subtract 20 both side,

⇒ 5x + 20 - 20 < 75 - 20

⇒ 5x < 55

⇒ 5x - 55 < 0

⇒ 5 (x - 11) < 0

⇒ x - 11 < 0

⇒ x < 11

Therefore, The required solution of inequality is,

⇒ x < 11

Learn more about the inequality visit:

https://brainly.com/question/25944814

#SPJ6

ASAP! GIVING BRAINLIEST! Please read the question THEN answer CORRECTLY! NO guessing. I say no guessing because people usually guess on my questions.

Answers

Answer:

The correct answer would be A

(I am not guessing I had the same quiz before)

HELP PLEASE SIMPLIFY !!!

Answers

Answer:

[tex]=x^{\frac{5}{6}}+2x^{\frac{7}{3}}[/tex]

Step-by-step explanation:

[tex]x^{\frac{1}{3}}\left(x^{\frac{1}{2}}+2x^2\right)\\\mathrm{Apply\:the\:distributive\:law}:\quad \:a\left(b+c\right)=ab+ac\\a=x^{\frac{1}{3}},\:b=x^{\frac{1}{2}},\:c=2x^2\\=x^{\frac{1}{3}}x^{\frac{1}{2}}+x^{\frac{1}{3}}\cdot \:2x^2\\=x^{\frac{1}{3}}x^{\frac{1}{2}}+2x^2x^{\frac{1}{3}}\\\mathrm{Simplify}\:x^{\frac{1}{3}}x^{\frac{1}{2}}+2x^2x^{\frac{1}{3}}:\quad x^{\frac{5}{6}}+2x^{\frac{7}{3}}\\x^{\frac{1}{3}}x^{\frac{1}{2}}+2x^2x^{\frac{1}{3}}\\x^{\frac{1}{3}}x^{\frac{1}{2}}=x^{\frac{5}{6}}[/tex]

[tex]x^{\frac{1}{3}}x^{\frac{1}{2}}\\\mathrm{Apply\:exponent\:rule}:\quad \:a^b\cdot \:a^c=a^{b+c}\\x^{\frac{1}{3}}x^{\frac{1}{2}}=\:x^{\frac{1}{3}+\frac{1}{2}}\\=x^{\frac{1}{3}+\frac{1}{2}}\\\mathrm{Join}\:\frac{1}{3}+\frac{1}{2}:\quad \frac{5}{6}\\\frac{1}{3}+\frac{1}{2}\\\mathrm{Least\:Common\:Multiplier\:of\:}3,\:2:\quad 6\\Adjust\:Fractions\:based\:on\:the\:LCM\\=\frac{2}{6}+\frac{3}{6}[/tex]

[tex]\mathrm{Since\:the\:denominators\:are\:equal,\:combine\:the\:fractions}:\quad \frac{a}{c}\pm \frac{b}{c}=\frac{a\pm \:b}{c}\\=\frac{2+3}{6}\\\mathrm{Add\:the\:numbers:}\:2+3=5\\=\frac{5}{6}\\=x^{\frac{5}{6}}\\2x^2x^{\frac{1}{3}}=2x^{\frac{7}{3}}\\=x^{\frac{5}{6}}+2x^{\frac{7}{3}}[/tex]

The value of m compared to the standard is
1/1000
1000
1/100
10

Answers

Idk either but it looks like the reciprocal so I would guess 1/m

Answer the inequality

Answers

Answer:

A.

Step-by-step explanation:

Add 4:

-5x ≤ 10

Divide by -5:

x ≥ -2

How many tons is 22,000 pounds?

Answers

Answer:

1 ton = 2,000 pounds

Step-by-step explanation:

With that said, 22,000 pounds is 11 tons because 2,000 x 11 = 22,000.

So 22,000 pounds is 11 tons.

Hope it helps and pls mark me brainliest if it did! :)

Mr. Hobbs took out a loan of $12,000 for 4 years at a simple annual
interest rate of 7%. How much interest did he pay?

Answers

Answer:

Step-by-step explanation:

I= P*r*t

I = 12000* 7% *4

Total interest paid was $3360.

Answer: $3,360

Step-by-step explanation:

Is 3/5 A.irrational, B.rational, C.natural and whole, or D.natural, whole integer and rational

Answers

Answer:

B

Step-by-step explanation:

3/5 is a fraction, meaning it isn't irrational, natural, whole or an integer, therefore the answer is rational (B).

Answer:

B.rational

Step-by-step explanation:

3/5 is written as a fraction so it is a rational number

It is not a whole number since it is a reduced fraction that is less than 1

Other Questions
Can someone plz help me out! With this problem I really need help ASAP! I will mark you as brainiest! Help help ASAP! The bending of which body part involves the working together of parts of the skeletal, muscular, and nervous systems? Compare Olmec civilization with one of the early civilizations in Asia/North Africa how do problems of evidence complicate the comparison Determine if the following salt is neutral, acidic or basic. If acidic or basic, write the appropriate equilibrium equation for the acid or base that exists when the salt is dissolved in aqueous solution. If neutral, write only NR. LiNO Records on a fleet of trucks reveal that the average life of a set of spark plugs is normally distributed with a mean of 22,100 miles. The fleet owner purchased 18 sets and found that the sample average life was 23,400 miles; the sample standard deviation was 1,412 miles. a) To decide if the sample data support the company records that the spark plugs average 22,100 miles, state your decision in terms of the null hypothesis. Use a 0.05 level of significance. b) What is the critical value for the test using a 0.05 level of significance? c) What is the test statistic? d) What is your decision? What is the natural resource rich area of northeastern China that japan invaded in 1931 and later became a colony to japanPlz I need this In 1970, 59% of college freshmen thought that capital punishment should be abolished; by 2005, the percentage had dropped to 35%. Is the difference real, or can it be explained by chance ASAP Mario was given the following enlargement. Side A corresponds to a side with length 24 centimeters. A side with length 6 centimeters corresponds with a side with length 12 centimeters. Figures not drawn to scale. What is the length of side A, in centimeters? 12 15 24 33 What happens to rib muscles and the diaphragm when youinhale and hold your breath? Can someone help me on these two questions? Arsha predicted that she would sell 225 magnets. She actually sold 240 magnets. What are the values of a and b in the table below? Percent Error Item Approximate value Exact value Error Absolute error Ratio Percent error Magnets 225 240 a b a = Negative StartFraction 15 over 225 EndFraction; b = negative 6.7 percent a = Negative StartFraction 15 over 240 EndFraction; b = negative 6.25 percent a = StartFraction 15 over 240 EndFraction; b = 6.25 percent a = StartFraction 15 over 225 EndFraction; b = 6.7 percent For a reaction to occur what must happen to the energy in order to break the chemical bond Which of the following was a direct result of Islamic expansion into spainA) Christians were forced to convert to islamB) the Umayyad army marched into central Asia C) Charles martel led armies in the battle at Poitier in 732.D) the Islamic law was changed to be more like catholicism please answer asap! A couple plans to have 5 children. The gender of each child is equally likely. Design a simulation involving 125 trials that you can use to model the genders of the children. Write your answers as numbers. Can you please help a strong statement to end essay that talks about why students should be allowed to use their cell phones during class time Which phrases from the excerpt include loaded words intended to evoke a sense of fear? Select two options. One angle of a right triangle measures 51 degrees. What is the measure of the other small angle? find the value of each trigonometric ratio part 3 Over the past ten years, the town's population doubled in size. The population is currently 12,000. What was the population ten years ago? Which of the following represents the given word problem if "P" represents the population ten years ago? 2(P + 10) = 12,000 2P = 12,000 P = 2(12,000)